Trả lời bởi giáo viên

Đáp án đúng: a

Ta có:

\(\)\(\)\(\begin{array}{l}{202^{303}} = {202^{3.101}} = {\left( {{{202}^3}} \right)^{101}}\\{303^{202}} = {303^{2.101}} = {\left( {{{303}^2}} \right)^{101}}\end{array}\)

Ta so sánh \({202^3}\) và \({303^2}\)

\(\begin{array}{l}{202^3} = {\left( {2.101} \right)^3} = {2^3}{.101^3} = {2^3}{.101^{1 + 2}} = {2^3}{.101.101^2} = {8.101.101^2} = {808.101^2}\\{303^2} = {\left( {3.101} \right)^2} = {3^2}{.101^2} = {9.101^2}\end{array}\)

Vì \(9 < 808\) nên \({9.101^2} < {808.101^2}\) hay \({303^2} < {202^3}\)

Do đó \({\left( {{{303}^2}} \right)^{101}} < {\left( {{{202}^3}} \right)^{101}}\)

Vậy \({303^{202}} < {202^{303}}\) .

Hướng dẫn giải:

Sử dụng các quy tắc để biến đổi hai lũy thừa hoặc cùng cơ số hoặc cùng số mũ và sử dụng quy tắc:

+) Nếu \(n < m\) thì \({a^n} < {a^m}\left( {a > 1;m,n \in N} \right)\)

+) Nếu \(a < b\) thì \({a^n} < {b^n}\left( {a,b \in \mathbb{N};n \in \mathbb{N}^*} \right)\)

Câu hỏi khác